Large Intestine Multiple choice Questions & Answers

Posted On:February 13, 2019, Posted By: Latest Interview Questions, Views: 2235, Rating :

Best Large Intestine Interview Questions and Answers

Dear Readers, Welcome to Large Intestine Objective Questions and Answers have been designed specially to get you acquainted with the nature of questions you may encounter during your Job interview for the subject of Large Intestine Multiple choice Questions. These Objective type Large Intestine Questions are very important for campus placement test and job interviews. As per my experience good interviewers hardly plan to ask any particular question during your Job interview and these model questions are asked in the online technical test and interview of many Medical Industry.

MCQs on Large Intestine

1. Which answers are true? In contrast to ulcerative colitis, Crohn's disease of the colon:

A.  Is not associated with increased risk of colon cancer.

B.  Seldom presents with daily hematochezia.

C.  Is usually segmental rather than continuous.

D.  Has a lower incidence of perianal fistulas.

E.  Never develops toxic megacolon.

Answer: BC

DISCUSSION: Crohn's disease of the colon is a patchy, segmental, chronic, transmural inflammatory process that penetrates the bowel wall to form fistulas but seldom causes rectal bleeding. In contrast, ulcerative colitis is a mucosal ulcerating process that extends continuously from the rectum to the more proximal colon and frequently bleeds. Both diseases can develop toxic megacolon, and both predispose the patient to increased risk of malignancy of the large intestine over the long term.

 

2. Which answers are true? Options to consider when operating for Crohn's disease of the large intestine include:

A.  Colectomy and ileorectostomy.

B.  Colectomy, closure of the rectal stump, and ileostomy.

C.  Colectomy and continent ileostomy (Kock pouch).

D.  Proctocolectomy and ileostomy.

E.  Proctocolectomy and ileal pouch–anal canal anastomosis.

Answer: ABD

DISCUSSION: Patients with colonic Crohn's disease who have minimal or mild rectal involvement can be treated by colectomy and ileorectostomy or by colectomy, closure of the rectal stump, and ileostomy. When severe rectal involvement is also present, proctocolectomy with permanent ileostomy is required. The Kock pouch (continent ileostomy) and the ileal pouch–anal canal operation are not performed for Crohn's disease because of the risk of recurrence of Crohn's disease in the ileal pouch in the postoperative period.

 

3. Crohn's disease:

A.  Is caused by Mycobacterium paratuberculosis.

B.  Is more common in Asians than in Jews.

C.  Tends to occur in families.

D.  Is less frequent in temperate climates than in tropical ones.

E.  Is improved by smoking.

Answer: C

DISCUSSION: The cause of Crohn's disease is unknown. No specific microorganism has been identified as a pathogen, and no clear-cut environmental factor, such as smoking, has been implicated, even though many patients with Crohn's disease are heavy smokers. The disease does tend to occur in families. It is more common among Jews than Asians and among people who live in temperate climates than those in tropical ones.

 

4. Recurrence after operation for Crohn's disease:

A.  Occurs after operations for ileal Crohn's but not colonic Crohn's.

B.  Is usually found just proximal to an enteric anastomosis.

C.  Rarely requires reoperation.

D.  Occurs in 1% of patients at risk per year during the first 10 years after the operation.

E.  Is prevented by maintenance therapy with corticosteroids.

Answer: B

DISCUSSION: Recurrence after operation for Crohn's disease often occurs just proximal to an enteric anastomosis or stoma and occurs at a rate of about 6% per year over the first 10 years after operation. Recurrence follows operations for both ileal and colonic Crohn's and is not prevented by medical therapy using corticosteroids. Reoperation is required for 30% to 50% of subjects at risk.

 

5. Excision rather than bypass is preferred for surgical treatment of small intestinal Crohn's because:

A.  Excision is safer.

B.  Bypass does not relieve symptoms.

C.  Excision cures the patient of Crohn's disease but bypass does not.

D.  Fewer early complications appear with excision.

E.  The risk of small intestine cancer is reduced.

Answer: E

DISCUSSION: Bypass of segments of small bowel affected with Crohn's disease is a safe operation with few complications, and one that usually relieves symptoms promptly. It leaves diseased bowel behind, however, which can flare in the future and can develop carcinoma. Excision, though it does not cure the Crohn's disease, removes the areas of severe involvement and so eliminates the risk of developing cancers in these segments.

 

6. Which statements about anorectal Crohn's disease are true?

A.  It may be the only overt manifestation of Crohn's disease.

B.  It accompanies large intestine Crohn's more often than small-intestine Crohn's.

C.  It subsides when associated small intestinal Crohn's is excised.

D.  It should not be treated operatively.

E.  It may subside in response to metronidazole, 250 mg. q.i.d.

Answer: ABE

DISCUSSION: Anorectal Crohn's disease may be the sole gross manifestation of Crohn's disease. It more often accompanies large-intestinal Crohn's than small-intestinal Crohn's. When present with small-intestinal Crohn's, resection of the small-intestinal disease does not affect the course of the anorectal disease. The anorectal disease may subside with metronidazole therapy alone, but local conservative therapy, such as draining abscesses or unroofing anal fistulas, may also relieve symptoms and promote healing.

 

7. The most common indication for operation in Crohn's disease of the colon is:

A.  Obstruction.

B.  Chronic debility.

C.  Bleeding.

D.  Perforation.

E.  Carcinoma.

Answer: B

DISCUSSION: Crohn's disease of the colon usually leads to operation because of chronic debility and inanition unresponsive to medical therapy. Obstruction, perforation, and bleeding are uncommon complications of colonic Crohn's. While for persons with Crohn's colitis the risk of carcinoma of the colon is four to six times that of a healthy control population, the presence of cancer in the colon is an unusual cause for operation for Crohn's colitis. In fact, most patients with Crohn's have their colons excised before sufficient time has elapsed for cancers to appear. Cancers usually do not appear until 10 years or more after the onset of disease.

 

8. Which of the following statements about surgical anatomy of the colon and rectum is/are correct?

A.  The cecum has the largest inner diameter of all segments of the colon (13 to 15 cm.).

B.  The rectosigmoid junction is situated at approximately 15 to 18 cm. from the anus.

C.  The rectum is entirely an intraperitoneal organ.

D.  The ileocolic, right colic, and middle colic arteries are branches from the inferior mesenteric artery.

E.  The arterial arcade created by communicating vessels at 1 to 2 cm. from the mesenteric is called the artery of Drummond.

Answer: BE

DISCUSSION: The cecum has, indeed, the largest inner diameter of all segments of the colon; however it usually measures between 7 and 9 cm. In patients with a distal obstruction and a continent ileocecal valve the cecum dilates. A diameter of 13 cm. or more is considered at high risk of perforation, and therefore surgery is often needed. The rectosigmoid junction is situated approximately 15 to 18 cm. from the anus, which places lesions in the sigmoid colon within reach of a rigid sigmoidoscope. The rectum is covered by peritoneum only in the anterior and superior aspect. The remainder of the rectum, or most of it, is extraperitoneal. The ileocolic, right colic, and middle colic arteries are branches from the superior mesenteric artery and supply blood to the ascending colon and the right half of the transverse colon. The artery of Drummond is an arcade that runs 1 to 2 cm. from the mesenteric border of the entire colon and provides communication between the superior mesenteric and inferior mesenteric arteries.

 

9. Which of the following statements about surgical procedures on the colon and rectum is/are correct?

A.  Successful healing of colonic anastomoses depends on the adequacy of the blood supply.

B.  In excising part of the colon containing cancer, the lymphatics should be avoided by dividing the mesentery close to the wall of the colon.

C.  Despite complete removal of the colon and rectum, transanal fecal flow can be preserved by means of an ileal pouch–anal anastomosis.

D.  When a colostomy is created it cannot be reversed.

E.  Colostomy can be life saving in patients with colonic perforation or obstruction.

Answer: ACE

DISCUSSION: Healing of colonic anastomoses depends on the adequacy of blood supply, which in turn depends on the tension on the anastomosis. Oncologic principles for surgery of colon cancer dictate that the mesentery be divided as close as possible to the origin of the blood vessels, to include the lymphatic vessels and nodes draining that area. In patients at risk for colon cancer, such as those with ulcerative colitis or familial polyposis, the construction of an ileal pouch–anal anastomosis allows for transanal fecal flow despite complete excision of the colon and rectum. Colostomies can often be life saving, especially in patients with colon perforation or obstruction, and are usually reversed unless the patient requires abdominoperineal resection of the rectum for cancer.

 

10. Which of the following statements about colon physiology is/are correct?

A.  Colonic recycling of urea is accomplished by the splitting of urea by bacterial ureases.

B.  Fermentation by colonic bacteria may rescue malabsorbed carbohydrates.

C.  The preferred fuel of the colonic epithelium is glucose.

D.  Absorption by the colonic mucosa is a passive process.

E.  Insoluble fibers create bulk in the stool.

Answer: ABE

DISCUSSION: One of the functions of the colon is to recycle nutrients used in the digestive process, such as bile salts, water, and electrolytes. Urea reaching the colon via either the ileal effluent or the mucosal circulation is split by bacterial ureases. The reabsorbed ammonia is returned to the liver, where it is used for amino acid and protein synthesis. Fermentation is the anaerobic process by which bacteria can degrade carbohydrates and proteins. The normal substrate for colonic fermentation is dietary fiber, which resists degradation by alpha-amylases in the small intestine. Starch polysaccharides are normally degraded by amylases and absorbed in the small intestine; however, when starch polysaccharides are not adequately degraded and absorbed, they can also be fermented and the caloric value recovered as short-chain fatty acids. n-Butyrate, one of the short-chain fatty acids produced by bacterial fermentation, is the preferred fuel of the colonic epithelium. The colonic epithelium utilizes n-butyrate as a fuel for the absorption of sodium and water. Insoluble fibers retain water and are poorly fermented by bacteria, thus producing fecal bulk.

 

11. Which of the following statements about colonic motility is/are true?

A.  Mass contractions involve only the rectum.

B.  “Antiperistaltic” contractions occur in the descending colon.

C.  The rectum can accommodate stool by receptive relaxation.

D.  Stool in the colon is propelled by tonic contractions.

E.  Defecation involves both sensory and motor pathways.

Answer: ADE

DISCUSSION: Various patterns of motility are present in the colon. In the ascending colon “antiperistalsis” waves generate retrograde flow of colonic contents back to the cecum. In the descending colon contents are propelled caudad by tonic contractions, separating them into a series of globular masses. A third type of contraction, called mass peristalsis, occurs at varying intervals through the entire colon's length. Finally, the rectum will undergo receptive relaxation to accommodate stool until defecation takes place. Normal defecation involves the perception and discrimination of stool and gas in the rectum, deferment of stool elimination until a socially acceptable time, expulsion of stool, and sealing of the anal canal. These processes depend on various sensory and motor pathways.

 

 

12. Which of the following statements about diagnostic studies for the colon and rectum is/are true?

A.  Acetylcholinesterase staining of rectal biopsies is unreliable for the diagnosis of Hirschsprung's disease.

B.  Cinedefecography is useful for detecting “hidden” prolapse or rectal intussusception.

C.  A negative osmotic gap in stool is indicative of secretory diarrhea.

D.  A colonic transit time study involves serial abdominal x-rays after ingestion of radiopaque markers.

E.  Carcinoembryonic antigen (CEA) is useful for monitoring patients after resection for colon cancer.

Answer: BCDE

 

DISCUSSION: Rectal biopsy should be performed in all patients with Hirschsprung's disease. The biposy specimen is examined for ganglion cells, but a much more reliable method is staining acetylcholinesterase, which has an accuracy rate of 99%. Cinedefecography involves instilling contrast medium into the rectum and recording the act of defecation with static radiographs and videofluoroscopy videotape. While the static radiographs allow measurement of the anorectal angle and the relationship between the anus and the coccyx/pubis plane, the videotape is used to detect rectal intussusception, which can be missed in static radiographs. In equivocal cases differentiation between malabsorptive and secretory diarrhea can be made by measurement of electrolytes and osmolality in stool. The osmotic gap is calculated by subtracting 2 × (Na + K) from the measured osmolality. A negative osmotic gap is indicative of secretory diarrhea, whereas a positive osmotic gap indicates malabsorptive diarrhea. In a colonic transit study, 20 radiopaque barium-impregnated markers are ingested before breakfast and plain films of the abdomen are obtained on days 4 and 6. The value of CEA is controversial, but most surgeons agree that it is helpful in monitoring following resection for colon cancer.

 

 

13. Which of the following statements about anorectal functional testing is/are true?

A.  Anorectal manometry is often performed through open-tipped multilumen catheters perfused with fluid.

B.  Anorectal manometry can differentiate between segmental and global defects of the anal sphincter in patients with incontinence.

C.  Electromyography can demonstrate persistent contraction of the pubis rectalis muscle during defecation, which is diagnostic of paradoxical pelvic floor contraction.

D.  Measurement of sensory thresholds may reveal insensitivity in patients with chronic constipation.

Answer: ABCD

 

DISCUSSION: Anorectal manometry is most commonly performed through open-tipped multilumen catheters perfused with fluid. These catheters are connected to a transducer and register internal and external sphincter pressures and presence of the anorectal inhibitory reflex. Anorectal manometry is particularly useful in patients with fecal incontinence since it can differentiate between segmental and global defects of the anal sphincter. Electromyography records the action potential derived from the different muscles involved in defecation through endoscopically placed mucosal electrodes along the colon. Electromyography can demonstrate persistent contraction of the pubis rectalis muscle during defecation, which is diagnostic of paradoxical pelvic floor contraction. Sensory threshold measurements may disclose a high rectal threshold (insensitivity) in patients with idiopathic constipation, extent of denervation, or myopathy in the colon wall.

 

 

14. Which of the following statements about the microbiology of the colon is/are true?

A.  The colon contains no more bacteria than the stomach.

B.  The predominant bacteria in the colon are aerobic.

C.  Nearly one third of the dry weight of feces is bacteria.

D.  Common bacteria in the colon are Bacteroides, Bifidobacterium, and Enterobacterium species.

E.  The colonic microflora is relatively stable.

Answer: CDE

 

DISCUSSION: The number of bacteria in the colon approaches 10 12 colony-forming units per ml., which is as many as the space in the colonic lumen allows and many more than in the stomach. The colonic microflora is relatively stable with a predominance of anaerobic bacteria. Typical species are Bacteroides, Bifidobacterium, and Enterobacterium. Depending on the fiber content of the diet, bacteria make up a significant proportion of the dry weight of feces. In the western world this can be as much as one third of the dry weight of feces.

 

 

15. Which of the following statements about bowel preparation for colon surgery is/are true?

A.  Bowel preparation is accomplished by a combination of mechanical cleansing and nonabsorbable antibiotics.

B.  Three days of clear liquids provides sufficient mechanical cleansing.

C.  Commercial electrolyte-polyethylene glycol solutions provide mechanical cleansing without inducing electrolyte imbalance.

D.  Nonabsorbable antibiotics such as neomycin and erythromycin base are administered the day before the operation in three doses.

E.  Intravenous antibiotics are also administered the day before surgery.

Answer: ACD

 

DISCUSSION: Bowel preparation is accomplished by a combination of mechanical cleansing and nonabsorbable antibiotics. In the past, 3 days of clear liquids was used as a means of mechanical cleansing; this was neither sufficient nor practical. Presently, an electrolyte-polyethylene glycol solution is commercially available that provides effective mechanical cleansing without inducing electrolyte imbalance. Nonabsorbable antibiotics such as neomycin and erythromycin base are administered the day before the operation, usually at 1 P.M., 2 P.M., and 11 P.M. Intravenous antibiotics are administered within hours of surgery to achieve maximal levels in the tissues during the operation.

 

 

16. Which of the following patients generally does not require surgical intervention as a consequence of acute diverticulitis?

A.  A 35-year-old man with no history of diverticulitis.

B.  A 68-year-old man status 2 weeks post–renal transplantation.

C.  A 55-year-old woman with hypertension and diabetes mellitus.

D.  A 50-year-old man with pneumaturia.

E.  A 46-year-old man with right-sided diverticulitis.

Answer: C

 

DISCUSSION: The majority of patients with diverticular disease are elderly and often have comorbid illnesses. The prognosis in these patients depends on the severity of the underlying inflammatory lesion. Certain subsets of patients, however, have been identified whose overall prognosis is worse. Patients younger than 40 years have a higher incidence of complications, as about 70% eventually require surgical intervention. Patients undergoing renal transplantation are routinely immunosuppressed. Such patients do not manifest the usual signs and symptoms of an inflammatory response. Delays in diagnosis and failure of the normal immune response mandates surgical intervention in virtually all of these patients. The presence of pneumaturia is strongly suggestive of a colovesical fistula. All such fistulas require resection of the diseased colon and repair the involved bladder. Patients with right-sided diverticulitis are usually misdiagnosed as acute appendicitis and, therefore, often are not diagnosed until laparotomy.

 

 

17. The test with the highest diagnostic yield for detecting a colovesical fistula is:

A.  Barium enema.

B.  Colonoscopy.

C.  Computed tomography (CT).

D.  Cystography.

E.  Cystoscopy.

Answer: E

 

DISCUSSION: Of the tests listed above, cystoscopy provides the highest diagnostic yield, between 80% and 95%. The most common finding on cystoscopy is localized inflammation and bullous edema of the bladder mucosa. Actual demonstration of the fistula is unusual, no matter which test is utilized. Barium enema usually demonstrates some abnormality; however, precise delineation of the fistula occurs in only 30% of cases. Recently, CT has been shown to be useful in diagnosing colovesical fistula with accuracy approaching that of cystoscopy. The low diagnostic yields (20%) of cystography and colonoscopy have limited their use.

 

 

18. Which of the following is not true of diverticular disease:

A.  It is more common in the United States and Western Europe than in Asia and Africa.

B.  A low-fiber diet may predispose to development of diverticulosis.

C.  It involves sigmoid colon in more than 90% of patients.

D.  Sixty per cent develop diverticulitis sometime during their lifetime.

E.  It is the most common cause of massive lower gastrointestinal hemorrhage.

Answer: D

 

DISCUSSION: Among all patients with diverticular disease only 20% can be expected ever to develop symptoms related to their disease. The development of diverticular disease has been linked to low-fiber diets, a type of diet more common in industrialized countries such as the United States and Western Europe. This correlates with the increased prevalence of diverticular disease in these regions. Diverticular hemorrhage accounts for 50% to 60% of all cases of massive lower gastrointestinal hemorrhage.

 

 

19. The most common indication for surgery secondary to acute diverticulitis is:

A.  Abscess.

B.  Colonic obstruction.

C.  Colovesical fistula.

D.  Free perforation.

E.  Hemorrhage.

Answer:A

 

DISCUSSION: Complications of diverticular disease include obstruction, fistulization, hemorrhage, and infection. By far, the most common indication for surgery is intra-abdominal abscess formation, accounting for 40% to 50% of all complications of diverticulitis. Intestinal obstruction accounts for another 10% to 30%, while free perforation can be expected in 10% to 15% of complicated cases of diverticulitis. Fistulization is the least common problem, occurring in only 4% to 10% of complicated cases. Bleeding from diverticula occurs in the complete absence of inflammation.

 

 

20. Which of the following is/are true about colorectal polyps?

A.  Familial juvenile polyposis is associated with an increased incidence of colon cancer.

B.  Although the propensity for development of malignancy is related to the size of a neoplastic polyp, those with mixed tubulovillous histologic appearance are most likely to develop malignant changes.

C.  The loss of a single tumor suppressor gene such as p53 is sufficient to lead to the development of malignancy in colorectal neoplastic polyps.

D.  Endoscopic polypectomy results in a decreased incidence of carcinomas of the colon and rectum.

Answer: AD

 

DISCUSSION: Juvenile polyps are hamartomas, and can  cause symptoms in children such as bleeding, obstruction, and intussusception. Familial juvenile polyposis is associated with increased risk of colon carcinoma. Polyps with mixed tubular and villous appearance (tubulovillous adenomas) have an intermediate risk of malignancy; villous adenomas are the most likely to contain malignancy in each size range. While p53 and other tumor suppressor genes may be associated with the adenoma-to-carcinoma transition, it appears likely that multiple genetic defects are involved in this transformation. Alterations in p53 appear to be among the last, as changes are uncommon in adenomas but very common in carcinomas. The National Polyp Study Group (USA) demonstrated that colonoscopic polypectomy does in fact reduce the incidence  of subsequent colorectal carcinomas, which supports the concept that most carcinomas begin as polyps and supports aggressive endoscopic removal.

 

 

21. Which of the following statements about familial adenomatous polyposis (FAP) is/are true?

A.  Inherited in an autosomal-dominant manner, this genetic defect is of variable penetrance, some patients having only a few polyps whereas others develop thousands.

B.  The phenotypic expression of the disease depends mostly on the genotype.

C.  Appropriate surgical therapy includes total abdominal colectomy with ileorectal anastomosis and ileoanal pull-through with rectal mucosectomy.

D.  Panproctocolectomy with ileostomy is not appropriate therapy for this disease.

E.  Pharmacologic management of this disease may be appropriate in some instances.

Answer: C

 

DISCUSSION: The genetic defect is of high penetrance: nearly all affected patients develop hundreds to thousands of polyps. By definition, at least 100 polyps must be present. Recent studies have shown that even patients with the identical point mutation can exhibit variability in the phenotypic expression, suggesting that environmental or other genetic factors play a significant role. The phenotypic variations concern age at onset, size of polyps, density of polyps, and extracolonic manifestations of the disease. Although panproctocolectomy with ileostomy is not well-accepted by patients because of the stoma, acceptable surgical options include panproctocolectomy with ileostomy, total colectomy with ileorectal anastomosis, and ileoanal anastomosis with rectal mucosectomy. No pharmacologic agents have been demonstrated to be efficacious in this condition, though several have been tried.

 

 

22. Which of the following statements about the etiology of chronic ulcerative colitis are true?

A.  Ulcerative colitis is 50% less frequent in nonwhite than in white populations.

B.  Psychosomatic factors play a major causative role in the development of ulcerative colitis.

C.  Cytokines are integrally involved in the pathogenesis of ulcerative colitis.

D.  Ulcerative colitis has been identified with a greater frequency in family members of patients with confirmed inflammatory bowel disease.

E.  Ulcerative colitis is two to four times more common in Jewish than in non-Jewish populations.

Answer: ACDE

 

DISCUSSION: Despite intensive investigation, the specific cause of ulcerative colitis remains unknown. There appears to be a clear genetic component involved in the etiology and distribution of ulcerative colitis. It is significantly less frequent in nonwhite than in white populations and significantly more frequent among Jews than among other populations. There is a strong familial concordance by disease category: the prevalence of inflammatory bowel disease is 10% to 25% in relatives of patients with confirmed Crohn's disease or ulcerative colitis. There is considerable uncertainty about the fundamental role of infectious agents in the primary pathogenesis of ulcerative colitis. Psychological factors may play a role in exacerbations of the disease, but they are not of primary importance in its pathogenesis. Recent studies have suggested that cytokines and other immunoregulatory substances are integrally involved in the pathogenesis of inflammatory bowel disease.

 

 

23. Surgical alternatives for the treatment of ulcerative colitis include all of the following except:

A.  Colectomy with ileal pouch–anal anastomosis.

B.  Left colectomy with colorectal anastomosis.

C.  Proctocolectomy with Brooke ileostomy or continent ileostomy.

D.  Subtotal colectomy with ileostomy and Hartmann closure of the rectum.

Answer: B

 

DISCUSSION: Ulcerative colitis is a mucosal inflammatory disease confined to the rectum and colon. It can thus be cured by total proctocolectomy. For that reason, the standard of therapy for many years was total proctocolectomy and ileostomy. In an effort to avoid permanent ileostomy a number of other alternatives have been evaluated, including subtotal colectomy with ileorectal anastomosis, proctocolectomy with continent ileostomy, and colectomy with endorectal ileal pouch–anal anastomosis. In the past, subtotal colectomy with ileorectal anastomosis was accepted as a compromise operation, with the knowledge that disease-bearing rectal tissue was retained. Because other definitive alternatives are currently available, ileorectal anastomosis is no longer appropriate for elective surgical treatment of ulcerative colitis. In an acutely ill patient or when the diagnosis is in question, subtotal colectomy with ileostomy and Hartmann closure of the rectum is the most expeditious choice and allows later restorative surgery. Partial colectomy has never been an acceptable alternative for elective operative management of ulcerative colitis; thus, left colectomy with colorectal anastomosis would not be an appropriate alternative.

 

 

24. The initial management of toxic ulcerative colitis should include:

A.  Broad-spectrum antibiotics.

B.  6-Mercaptopurine.

C.  Intravenous fluid and electrolyte resuscitation.

D.  Opioid antidiarrheals.

E.  Colonoscopic decompression.

Answer: AC

 

DISCUSSION: Toxic colitis is a potentially life-threatening complication of chronic ulcerative colitis. Typically it manifests clinically with the onset of abdominal pain and severe diarrhea, followed by abdominal distention and generalized tenderness. Once megacolon and toxicity develop, fever, leukocytosis, pallor, tachycardia, lethargy, and shock set in. The initial treatment for toxic megacolon thus includes intravenous fluid and electrolyte resuscitation, nasogastric suction, broad-spectrum antibiotics to provide anaerobic and aerobic gram-negative coverage, and total parenteral nutrition to improve nutritional status. Large intravenous doses of corticosteroids are generally administered to treat the colitis. In addition, many patients with toxic megacolon are already receiving steroid therapy and, so, need stress doses of steroids to prevent adrenal crisis. The immunosuppressive drugs 6-mercaptopurine and azathioprine may play a role in the management of refractory ulcerative colitis; however, these drugs are not indicated in the acute management of toxicity. Cyclosporine was shown to be effective in treating acute refractory ulcerative colitis in a single controlled trial, but this has not yet been confirmed by other prospective studies, and it remains a potentially dangerous drug. Opioid antidiarrheals should be avoided since they may exacerbate the colonic dilatation and increase the possibility of perforation. Limited proctoscopy may be helpful in determining the cause of the attack, but colonoscopy may be dangerous and is contraindicated in the face of acute toxic megacolon. If toxic colitis, with or without megacolon, does not improve within 48 hours, emergency surgery is warranted.

 

 

25. Which finding(s) suggest(s) the diagnosis of chronic ulcerative colitis as opposed to Crohn's colitis?

A.  Endoscopic evidence of backwash ileitis.

B.  Granulomas on biopsy.

C.  Anal fistula.

D.  Rectal sparing.

E.  Cobblestone appearance on barium enema.

Answer: A

 

DISCUSSION: It has become increasingly important to distinguish between ulcerative colitis and Crohn's colitis, since the operative therapy for the two disease processes is quite different. Patients with ulcerative colitis are candidates for colectomy with ileoanal anastomosis, whereas Crohn's disease is a clear contraindication to this operation. Clinical findings suggestive of Crohn's disease include anal fistula or other perianal disease, though it must be kept in mind that approximately 10% of patients with ulcerative colitis may also develop perianal problems secondary to their chronic diarrhea. Endoscopic or radiographic evidence of rectal sparing is powerful evidence against a diagnosis of ulcerative colitis. However, if patients have been treated with steroid or salicylate enemas, they may have less active disease in the rectum than in the more proximal colon, a finding that could mislead the clinician about the presence or degree of rectal involvement. The deep linear ulcers that lead to a cobblestone appearance on barium enema are strongly suggestive of Crohn's disease. Typically, ulcerative colitis is confined to the rectum and colon. Frank small bowel involvement is suggestive of Crohn's disease; however, patients with active pancolitis may have secondary inflammation of the ileum, which has been called backwash ileitis. This clears after colectomy. The differential diagnosis may ultimately rely on histologic evaluation. Endoscopic biopsies are not generally useful since they only sample 3-mm. deep segments of mucosa and submucosa. Transmural inflammation and granulomas on surgical pathologic specimens are pathognomonic of Crohn's disease.

 

 

26. An 80-year-old man who has been bedridden for many years following a stroke presents with acute onset of abdominal distention, obstipation, and colicky abdominal pain. Abdominal x-rays reveal dilated loops of small bowel and a dilated sigmoid colon resembling a bent inner tube. Examination reveals distention with mild direct tenderness but no rigidity or rebound tenderness. Initial management should consist of:

A.  Barium enema examination.

B.  Laparotomy with resection of descending colon and descending colostomy.

C.  Multiple cleansing enemas to remove impacted feces.

D.  Rigid sigmoidoscopy and decompression of the sigmoid colon.

Answer: D

 

DISCUSSION: This patient appears to have presented with the classic signs and symptoms of acute sigmoid volvulus. The majority of patients with colonic volvulus are elderly men with underlying neurologic dysfunction. They are commonly referred from a chronic care facility. The patient's condition should be assessed immediately. In the absence of peritonitis the preferred initial management consists of urgent endoscopic detorsion of the volvulus. In most patients this can be easily accomplished with a rigid 25-cm. sigmoidoscope or a flexible fiberoptic colonoscope. Often detorsion of the sigmoid colon results in a dramatic expulsion of gas and feces. A rectal tube should be introduced into the sigmoid and then taped to the thigh or buttock and left in place to prevent immediate recurrence of the volvulus.

 

 

27. Axial twisting of the right colon or cecal volvulus has been shown to be associated with each of the following except:

A.  A history of abdominal operation.

B.  A mobile cecum.

C.  An obstructing lesion in the transverse or left colon.

D.  Inflammatory bowel disease.

Answer: D

 

DISCUSSION: Volvulus of the right colon is less common than sigmoid volvulus and may involve either an axial twist of the right colon or a cephalad fold of the cecum (cecal bascule). A mobile cecum is a prerequisite for cecal volvulus and may occur in up to one third of individuals. Cecal volvulus has also been called postoperative volvulus because of its tendency to follow abdominal surgical procedures. Obstructing lesions in the distal colon may lead to distention and torsion of the right colon in patients with abnormalities of cecal fixation.

 

 

28. Sigmoid volvulus has been associated with each of the following except:

A.  Chronic constipation and laxative abuse.

B.  Chronic rectal proplapse.

C.  Chronic traumatic paralysis.

D.  Medical management of Parkinson's disease.

Answer: B

 

DISCUSSION: The development of sigmoid volvulus depends on the presence of a dilated, redundant sigmoid colon. This acquired redundancy may be secondary to long-term ingestion of a high-residue diet, particularly in parts of the world where the disease is common. In the United States, the most prominent association is chronic constipation and excessive reliance on laxatives or enemas. Other contributing factors include neurologic or psychiatric conditions such as Parkinson's disease, Alzheimer's disease, multiple sclerosis, traumatic paralysis, chronic schizophrenia, pseudobulbar palsy, and senility. Patients are frequently bedridden and are being managed with various neuropsychotropic drugs, both of which may alter bowel motility.

 

 

29. Which of the following statements is not true about inhereted susceptibility to colon cancer?

A.  There is no known genetic susceptibility to colon cancer.

B.  There are known genetic susceptibilities to colon cancer, but they are always associated with multiple adenomatous polyps.

C.  There are known genetic susceptibilities to colon cancer, but they are always associated with specific ethnic or racial groups.

D.  None of the above.

Answer: D

 

DISCUSSION: In addition to familial polyposis syndrome, hereditary nonpolyposis colorectal cancer (HNPCC) includes site-specific colorectal cancer and cancer family syndrome, which are synonymous with Lynch syndromes I and II, respectively. Autosomal-dominant inheritance of colorectal cancer is observed in both syndromes, and the average patient age at cancer diagnosis is 45 years. Persons with HNPCC have a high occurrence of synchronous and proximal colon malignancies. Affected persons usually exhibit one or several adenomatous polyps. The polyps on average are larger, have more villous histology, and occur at a younger age than adenomas in the general population. Cancer family syndrome also includes other malignancies, especially endometrial lesions but also ovarian, gastric, small intestinal, and renal cancer.

 

 

30. Which of the following recommendations for adjuvant chemotherapy of colorectal carcinoma are true?

A.  Patients with Stage I or Dukes A and B1 disease should receive adjuvant treatment for 1 year with levamisole combined with 5-FU.

B.  Patients with Stage III or Dukes C disease should receive adjuvant treatment for 1 year with levamisole combined with 5-FU.

C.  There is no role for adjuvant therapy for colon cancer at any stage.

D.  Adjuvant chemotherapy is active in colon cancer only when combined with radiotherapy.

Answer: B

 

DISCUSSION: Some 50% to 60% of patients with colorectal cancer have tumors that penetrate the serosa or involve the regional lymph nodes, eventually recur, and end fatally. Therefore, adjuvant therapy to improve the mortality was sought for this group of patients. 5-Fluorouracil (5-FU) is the most active drug used against colon cancer, but it achieves only a 10% to 20% response in patients with advanced disease. Levamisole is thought to be an immunomodulating agent in advanced colorectal carcinoma. Randomized controlled trials of 5-FU with levamisole, levamisole alone, and surgery in patients with Dukes B2 or C colon cancer were performed and demonstrated that levamisole plus 5-FU and levamisole improve disease-free survival for patients with Dukes B and C lesions. Subsequent analysis demonstrated that Dukes C patients receiving levamisole and 5-FU also had slightly prolonged survival.

A larger, confirmatory intergroup trial was launched that demonstrated that in patients with Dukes C carcinomas of the colon, adjuvant treatment for 1 year with levamisole combined with 5-FU reduced the risk of cancer recurrence by 41% and reduced mortality overall by 33%, but the results in patients with Dukes B2 disease was equivocal.

 

 

31. Optimal front-line treatment of squamous cell carcinoma of the rectum includes:

A.  Abdominal perineal resection.

B.  Low anterior resection when technically feasible.

C.  Radiation therapy.

D.  Chemotherapy.

E.  Combined radiation and chemotherapy.

Answer: E

 

DISCUSSION: Combination radiation therapy and chemotherapy is now the treatment of choice for squamous cell carcinoma of the anus. The area of the primary lesion is biopsied, and the patient begins radiotherapy to the pelvis. If inguinal lymph nodes are enlarged, they are also biopsied, usually by fine-needle aspiration, and if positive, they are included in the field of radiation.

Following radiation therapy, patients receive intravenous 5-FU and mitomycin C. Patients who fail therapy have limited options, including additional chemotherapy or radiotherapy. Salvage therapy may also include abdominoperineal resection (APR), lymphadenectomy, or a diverting colostomy, depending on the nature of the recurrence.

 

 

32. Which of the following statement(s) is/are true about the maintenance of continence?

A.  It depends on both the internal and external sphincters as well as the puborectalis.

B.  Resting pressure offers a high-pressure zone that increases resistance to the passage of stools.

C.  Maximal squeeze pressure can last no more than 1 minute.

Answer: ABC

 

DISCUSSION: Continence depends on numerous complex and interrelated anal, rectal, pelvic floor, and colon factors. Resting pressure depends primarily on the internal sphincter and serves to increase resistance to the passage of stool. Squeeze pressure, generated by contraction of the external sphincter, increases anal canal resting pressure and helps prevent leakage when the rectal contents are presented to the proximal anal canal at inopportune times. It lasts but a minute before fatiguing. The anorectal angle produced by anterior pull of the puborectalis encircles the rectum at the level of the anorectal ring and helps to maintain continence.

 

 

33. Which of the following statement(s) about complete rectal prolapse, or procidentia is/are true?

A.  Rectal prolapse results from intussusception of the rectum and rectosigmoid.

B.  The disorder is more common in men than in women.

C.  Continence nearly always is recovered after correction of the prolapse.

D.  All of the above are true.

Answer: A

 

DISCUSSION: Rectal prolapse is believed to be the result of intussusception of the rectum and rectosigmoid. The condition predominates in women, in those who strain excessively, and in those suffering from mental disorders. Pregnancy and delivery are not implicated, as the condition can be observed in men and in nulliparous women. By the time the diagnosis is established, 50% of patients are incontinent, and continence improves in only half of the patients after surgical correction of the prolapse.

 

 

34. Which of the following statements about hemorrhoids is/are not true?

A.  Hemorrhoids are specialized “cushions” present in everyone that aid continence.

B.  External hemorrhoids are covered by skin whereas internal hemorrhoids are covered by mucosa.

C.  Pain is often associated with uncomplicated hemorrhoids.

D.  Hemorrhoidectomy is reserved for third- and fourth-degree hemorrhoids.

Answer: C

 

DISCUSSION: Hemorrhoids are specialized, highly vascularized cushions in the anal canal that aid continence. The cause of hemorrhoids is unknown, but they may represent nothing more than the downward sliding of anal cushions associated with straining and irregular bowel habits. They are classified and treated according to the severity of symptoms. External hemorrhoids are covered with anoderm and are distal to the dentate line. Internal hemorrhoids are covered by the mucosal lining of the anal canal proximal to the dentate line. They can cause painless bleeding, usually in association with defecation. Uncomplicated hemorrhoids usually are not associated with pain, but fissures more often are. Hemorrhoidectomy is reserved for third-degree (bleeding with prolapse requiring manual reduction) and fourth-degree (permanently prolapsed with or without bleeding) hemorrhoids.

 

 

35. The widely accepted treatment of most localized epidermoid, cloacogenic, or transitional cell carcinoma of the anal canal is:

A.  Surgical resection.

B.  Chemotherapy alone.

C.  Radiotherapy alone.

D.  Combined chemoradiation.

Answer: D

 

DISCUSSION: Tumors arising in the anal canal or in the transitional zone that have squamous, basaloid, cloacogenic, or mucoepidermoid epithelium are similar in their clinical presentation and response to treatment. Combined chemoradiation (the so-called Nigro protocol) promises to preserve continence, avoid colostomy, and offer a similar survival rate. Local excision is reserved for the few very small and superficial lesions. For most lesions, chemoradiation—external-beam radiation, 5-fluorouracil, and mitomycin C—is the treatment of choice.

 

 

36. Which statement(s) is/are true about hidradenitis suppurativa?

A.  It is a disease of the apocrine sweat glands.

B.  It causes multiple perianal and perineal sinuses that drain watery pus.

C.  The sinuses do not communicate with the dentate line.

D.  The treatment is surgical.

E.  All of the above.

Answer: E

 

DISCUSSION: Hidradenitis suppurativa is an inflammatory process of the sweat glands characterized by abscess and sinus formation. The disease may involve other areas where apocrine glands are present, such as the axilla, mammary, inguinal, and genital regions. The affected areas have a blotchy, purplish appearance with numerous sinuses draining watery pus. The condition must be differentiated from cryptoglandular fistulas, which communicate with the dentate line, and Crohn's disease, which may track to the anorectum proximal to the dentate line. Treatment consists of unroofing sinuses for limited disease and wide local excision for more advanced disease.

 

 

37. Which of the following statements regarding the vasculature of the colon and rectum is/are correct?

 

a. The middle colic artery is a branch of the superior mesenteric artery

b. The inferior mesenteric artery supplies the descending and sigmoid segments of the colon

c. An complete anastomotic arcade paralleling the colon wall is present in only 15 to 205 of individuals

d. The middle colic artery is a branch of the inferior mesenteric artery

Answer: a, b, c

 

Two major arterial systems supply the colon. The right colon is predominantly supplied by the superior mesenteric artery. The major branches of this artery that perfuse the right colon include the ileocolic branch, which supplies the ileocecal junction; the right colic, which supplies the ascending colon; and the middle colic artery, which supplies the hepatic flexure and the transverse colon to its midpoint. The left colon is predominantly supplied by the inferior mesenteric artery, which derives its origin from the abdominal aorta. The distal transverse colon and the descending colon obtain their blood supply from the left colic branch of the inferior mesenteric artery, while the sigmoid colon obtains its blood supply from sigmoidal branches. The colic arteries bifurcate and form vascular arcades so that the resultant marginal Drummond artery forms an anastomosis between the superior mesenteric artery and the inferior mesenteric artery. However, considerable anatomic variation exists with respect to this arterial arcade, and a complete anastomosis is present in only 15% to 20% of people.

 

 

38. Which of the following agents have been proposed as sensory neurotransmitters for the colon?

 

a. Acetylcholine

b. Substance P

c. Calcitonin gene-related peptide

d. Bradykinin

e. Somatostatin

Answer: b, c

 

Sensory neurons within the wall of the colon perceive mechanical and chemical stimuli from the luminal contents. Their axons project both to motor neurons as well as to prevertebral ganglia and higher neural centers. Mediators of such sensory input appear to be substance P and calcitonin-gene related peptide. The role of sensory neurons in transmitting information remains uncertain, but since they have been shown to synapse on the excitatory and inhibitory motor neurons, they probably play an important role in modulating spontaneous contractions.

 

 

39. How much of the daily insensible water loss is due to loss in stool?

 

a. 200 ml

b. 400 ml

c. 600 ml

d. 800 ml

e. 1000 ml

Answer: a

 

Compared with the small intestine, the epithelium of the colon is relatively impermeable and requires a considerably longer period of time to absorb the salt, water, and carbohydrate presented to it. Between 500 and 1500 g of a semiliquid material enter the colon on a daily basis. Most of this substance is absorbed, yielding a stool output that weighs about 200 g. The reason for this efficient absorptive capacity, despite the relative impermeability of the colonic epithelium, relates to the close interrelation between epithelial transport and colonic motor activity. Transit through the large bowel takes about 10 times as long as that through the small intestine, usually requiring 2 or 3 days. Such slow passage provides optimal opportunity for the luminal contents to come into surface contact with the absorptive epithelium. Thus, sodium and water, which are extracted against high electrochemical and osmotic gradients, are efficiently absorbed. Further, the relatively static conditions of the colon allow the proliferation of vast numbers of anaerobic bacteria, which in turn break down carbohydrate to volatile fatty acids, which then are rapidly absorbed. The more rapid the colonic transit, the less efficient are these absorptive processes.

 

 

40. A 52-year-old woman is involved in an automobile accident and sustains an open fracture of the fight femur, compression fractures of the 10th and 11th thoracic vertebrae and right pulmonary contusion. On the fourth day after injury, her abdomen is noted to be distended, tympanitic and diffusely tender. Abdominal radiographs reveal gaseous distension of the ascending and transverse segments of the colon. The cecum is 13 cm in greatest diameter. Appropriate management includes which of the following as the next step?

 

a. Right hemicolectomy

b. Operative cecostomy

c. Colonoscopy

d. Contrast enema

e. Observation

Answer: c

 

Acute obstruction of the colon in the absence of any identifiable mechanical obstructing lesion is known as Ogilvie’s syndrome. Acute pseudo-obstruction usually involves the proximal colon, but may extend throughout the transverse colon and, rarely, the descending colon as well. The cause of this syndrome is unknown, but it almost always arises in critically ill or bedridden patients who have serious concomitant disease. The syndrome is occasionally seen in traumatized patients, particularly if a vertebral fracture is present. The hallmark of the disease is severe abdominal distention. Roentgenographic films of the abdomen show marked gaseous distention of the colon that is typically localized to the right colon.

The major problem with Ogilvie’s syndrome is the risk of cecal perforation. This risk is substantial, and the more prolonged the cecal distention, the greater this risk becomes. Although a considerable debate exists in the literature about the extent to which cecal dilatation can occur before perforation becomes a likely possibility, some type of decompressive procedure should be considered when the cecum reaches 10–12 cm in diameter. Fiberoptic colonoscopy is the treatment of choice and is initially successful in 90% or more of patients. In the event that colonoscopy does not work or is not readily accessible, operative cecostomy is the accepted alternative therapy.

 

 

41. Which of the following features is/are consistent with a diagnosis of colonic inertia?

 

a. Alternating episodes of severe constipation and normal bowel activity

b. Total bowel transit time of 24 hours

c. Total bowel transit time of 48 hours

d. Total bowel transit time of 96 hours

e. Marfinoid habitus

Answer: d

 

Colonic inertia, or slow transit constipation, is primarily a disease of young women. Despite attempts at initiating bowel movements with fiber supplementation, large doses of laxatives and enemas, normal bowel movement patterns are not established. The etiology of this condition is totally unknown, but a likely etiology is some aberration in the neurochemical control of the colon, possibly within the enteric nerves. Abnormalities within the neural elements of the myenteric plexus suggest that disturbances in neuromodulation of colonic motility may play a role in some patients. The treatment of colonic inertia has proved difficult and many patients have required subtotal colectomy to correct the severe constipation. Diagnosis of this condition is usually achieved by assessing colonic transit with various radio-opaque markers. After ingestion of such markers, sequential abdominal films are taken to assess movement of markers in each segment of the colon. Total transit time in normal subjects averages about 35 hours. Total transit time in excess of 72 hours is clearly abnormal.

 

 

42. Which of the following statements regarding the myenteric plexus of the colon is/are correct?

 

a. The myenteric plexus is located between the longitudinal and circular layers of the bowel wall

b. The myenteric plexus contains only sensory neurons

c. The density of neurons with the colonic plexuses decreases along the length of the bowel

d. Neurons of the myenteric plexus control the motor function of the colon

Answer: a, c, d

 

Similar to other regions of the gut, two groups of plexuses exist within the wall of the colon. The submucosal plexus is located between the muscularis mucosa and the circular muscle layer of the muscularis propria. Sandwiched between the circular muscle and the outer longitudinal muscle is the myenteric plexus. The myenteric plexus appears to be intimately involved with colonic motility. The plexus is composed of ganglia and clusters of nerve cell bodies that are linked together by bundles of nerve processes. Although the myenteric plexus is well developed throughout the entire length of the colon, the density of nerve cells is greatest in the proximal one third of the colon, similar to the density that exists throughout the small intestine. The physiologic significance of this organization remains to be defined, but probably contributes to the different motility patters that exist throughout the large bowel.

 

43. Which of the following ocular manifestations of ulcerative colitis respond to therapy with steroids or immunosuppressive agents?

 

a. Iridis

b. Uveitis

c. Retrobulbar neuritis

d. Ulcerative panophthalmitis

Answer: a, b, c

 

A number of ocular manifestations of ulcerative colitis exist. Included in this group are iridis, uveitis, conjunctivitis, episcleritis, retinitis, and retrobulbar neuritis. With the exception of ulcerative panophthalmitis, ocular symptoms are closely related to disease activity and respond to therapy with steroids or immunosuppressive agents.

 

 

44. Which of the following statements regarding the risk of cancer in the context of ulcerative colitis is/are correct?

 

a. After 10 years of active disease, the risk of cancer approximates 20% to 30%

b. After 10 years of active disease, the risk of cancer approximates 2% to 3%

c. The risk of colon cancer in ulcerative colitis is identical to controls

d. After 20 years of disease activity, the risk of colon cancer approximates 80%

Answer: b

 

Significant dysplasia or suspected colon cancer is a clear indication for colectomy in patients with ulcerative colitis. Earlier studies have suggested that the risk of cancer is relatively low for the first ten years after the onset of disease activity (roughly 2% to 3%). The incidence of colon cancer then begins to climb at a rate of 1% to 2% per year. By the time the patient has had ulcerative colitis for 20 years, the risk of colon cancer approximates 20%. Many epidemiologists believe that earlier studies overestimated the risk of malignancy due to referral bias and the imperfection of retrospective surveys performed in tertiary referral hospitals.

 

 

45. Which of the following features would be more consistent with Crohn’s disease than with ulcerative colitis?

 

a. Transmural inflammation

b. Microscopic evidence of granulomas within mucosal biopsies

c. Microscopic evidence of submucosal thickening and fibrosis

d. Microscopic evidence of submucosal inflammation

Answer: a, b, c, d

 

Transmural changes are found in Crohn’s disease of the colon in which all layers of the colonic wall may be involved in a granulomatous inflammatory process. In its earliest stages, the lesions consist of infiltration of round cells and polymorphonuclear leukocytes into the crypts of Lieberkühn at the base of the mucosa. Crypt abscesses are common. Microscopy reveals vacuolization of overlying epithelial cells, swelling of mitochondria, and widening of intercellular spaces. Submucosal thickening and fibrosis associated with recurrent submucosal inflammation are more common in Crohn’s disease than in ulcerative colitis.

 

 

46. A 19-year-old male is seen in consultation with complaints of bloody diarrhea (10 bowel movements per day), and weight loss (10 pounds). Physical examination reveals the presence of two circular, 4 cm erythematous lesions on the trunk. Each lesion has an area of necrosis in the center. The abdominal examination reveals mild hypogastric tenderness. The stool is guaiac positive. The most appropriate next diagnostic step includes which of the following?

 

a. Barium enema

b. Flexible sigmoidoscopy

c. Liver biopsy

d. Chest x-ray

Answer: b

 

The diagnosis of ulcerative colitis is one of exclusion. There are no definitive, laboratory, radiologic, or histologic features. All patients presenting with bloody diarrhea should have an infectious etiology excluded. Stool samples and biopsy specimens should be evaluated for Campylobacter, Salmonella, pathogenic E coli, amebic colitis, and C difficile. Flexible sigmoidoscopy is the first step in diagnosis, because ulcerative colitis involves the distal colon and rectum in 90% to 95% of the cases. Mild cases may show only a loss of normal vascular pattern, a granular texture, and microhemorrhages when the friable mucosa is touched by the endoscope. More advanced cases may demonstrate spontaneous bleeding, ulceration, and purulent exudate. Mucosal biopsies are essential in establishing the diagnosis. Lesions of the skin and oral cavity are frequently observed in patients with ulcerative colitis. Pyoderma gangrenosum is observed in approximately 1% of patients.

 

 

47. Which of the following statement(s) regarding ulcerative colitis is/are correct?

 

a. The most common age of onset for ulcerative colitis is in early adulthood

b. Approximately 25% of cases of ulcerative colitis occur after the age of 60

c. Males are affected approximately twice as frequently as females

d. Approximately 10% to 25% of patients with ulcerative colitis have first degree relatives with the disease

Answer: a, d

 

Most cases of ulcerative colitis have onset between the ages of 15 and 40 years. While the age of onset may extend to old age, only 3% to 5% of the cases have onset after age 60. Males and females are affected equally frequently. Clear-cut familial patterns have been observed in ulcerative colitis. Ten to 25% of patients with this disease have first degree relatives with ulcerative colitis. Monozygotic twins have higher concordance for inflammatory bowel disease than dizygotic twins. Geographic as well as racial differences influence the occurrence of the disease. There is currently no definitive evidence regarding the role of genetic and environmental influences in the determination of familial patterns.

 

 

48. Many patients with ulcerative colitis are operated upon electively with total abdominal colectomy, rectal mucosectomy, formation of a small intestinal reservoir, and ileoanal anastomosis. The most common postoperative complication after this operation is which of the following?

 

a. Enterocutaneous fistula

b. Small bowel obstruction

c. Pulmonary embolism

d. Urinary retention

Answer: b

 

The major postoperative morbidity following restorative proctocolectomy is small bowel obstruction. Bowel obstruction rates requiring re-operation has been reported in 10% to 20% of most series of patients undergoing ileal pouch-anal anastomosis. Pelvic and wound infections have been reported to occur in 10% of patients undergoing ileoanal anastomosis, although the overall infection rate has been reduced to approximately 5% in several recent series. Conversion to a permanent ileostomy as a result of postoperative complications is observed in fewer than 5% of patients.

 

 

49. A 25-year-old woman with known ulcerative colitis presents to the emergency room with a 24-hour history of abdominal pain, distention, and obstipation. Physical examination reveals a temperature of 38.6° C, abdominal distention, and diffuse abdominal tenderness. Abdominal x-rays show marked colonic dilatation, most pronounced in the transverse colon. Laboratory examination reveals a white blood count of 19,000/mm3. Over the first 24 hours of hospitalization, symptoms are progressive in spite of intravenous fluid resuscitation, nasogastric suctioning, and intravenous antibiotics. The most appropriate management for this patient would include which of the following?

 

a. Decompressive colonoscopy

b. Proctocolectomy with formation of end ileostomy

c. Total abdominal colectomy with formation of Hartmann pouch and end ileostomy

d. Cecostomy

Answer: c

 

Acute toxic megacolon occurs in 6% to 13% of patients with ulcerative colitis. Initial treatment for toxic megacolon includes intravenous fluid and electrolyte resuscitation, nasogastric suctioning, broad spectrum antibiotics, and total parenteral nutrition. The therapeutic role of intravenous steroids in toxic megacolon is controversial. Most patients presenting with a severe attack of ulcerative colitis are already receiving steroid therapy and require stress doses of corticosteroids to prevent adrenal crisis. When symptomatology is progressive or when there is evidence of colonic perforation, emergency surgery is indicated.

Postoperative complications including sepsis, wound infection, intraperitoneal abscess, fistula formation, and delayed wound healing are common and have been reported in up to 50% of patients. The presence of colonic perforation doubles operative risk. In the presence of toxic megacolon or colonic perforation, the operation should be definitive without being overly aggressive. Abdominal colectomy with ileostomy and Hartmann closure of the rectum is the procedure of choice. After recovery, delayed surgery for restoration of continence can be performed. Leaving the rectum intact allows its use for subsequent mucosal proctectomy and ileoanal anastomosis.

 

 

50. The most common postoperative complication after formation of a continent ileostomy (Kock pouch) is which of the following?

 

a. Nipple valve failure

b. Small bowel obstruction

c. Pancreatitis

d. Ischemic necrosis of the pouch

Answer: a

 

The continent ileostomy has been associated with a high postoperative complication rate. Most complications are related to displacement of the nipple valve, producing fecal incontinence and difficulty in intubating and emptying the pouch. Valve failure has been reported to occur in up to 40% of patients. Ten to 20% of patients suffer postoperative bowel obstructions. Several syndromes of ileostomy dysfunction related to Cock pouch have also been reported. These are variably described as stagnant loop syndrome, pouchitis, and nonspecific ileitis. Clinical features include diarrhea, malabsorption of fat and vitamin B12, proliferation of anaerobic bacteria, inflammation of the pouch, and incontinence. Crohn’s disease is a clear contraindication to performing this operation, as the rate of postoperative complication is much higher in this group.

 

 

51. One year following ileal pouch-anal anastomosis, the mean 24-hour stool frequency is which of the following?

 

a. Two to three

b. Five to six

c. Eight to nine

d. Eleven to twelve

Answer: b

 

Overall, mean 24-hour stool frequencies average five or six bowel movements in the late follow-up period after ileal pouch-anal anastomosis.

 

 

52. A 30-year-old male two years postoperative after total abdominal colectomy with ileoanal anastomosis reports a sudden increase in stool frequency, nocturnal leakage, and low-grade fevers. Physical examination is unremarkable. Flexible endoscopic examination of the small intestinal pouch reveals a friable erythematous mucosa. Biopsies of the mucosa are obtained. While awaiting biopsy results, which of the following is the most appropriate empiric therapy?

 

a. Oral corticosteroids

b. Oral vancomycin

c. Oral metronidazole

d. Corticosteroid enema

Answer: c

 

Nonspecific enteritis or pouchitis is the most common late complication of ileal pouch-anal anastomosis, occurring in as many as 15% of patients. The clinical symptoms include high stool frequency, watery stools, fat malabsorption, urgency, nocturnal leakage, and rectal bleeding. Patients may manifest fever, malaise, and arthralgias. The etiology of this condition is unknown. Most patients respond to treatment with metronidazole.

 

 

53. A 72-year-old man returns to the hospital 2 weeks following right hip arthroplasty with complaints of 48 hours of mucoid diarrhea, fever, and crampy abdominal pain. Physical examination reveals dehydration, diffuse abdominal tenderness, and a temperature of 102°F. Outpatient medications have included digoxin, propranolol, and cephalothin. If antibiotic-associated colitis is suspected, which of the following is/are appropriate diagnostic tests?

 

a. Fecal leukocyte smear

b. Stool culture for C difficile

c. Measurement of C difficile toxin in stool

d. Barium enema

e. Computed topography of abdomen

Answer: a, b, c

 

The fecal leukocyte test is a simple, rapid screening measure that is sometimes useful in supporting the diagnosis of C difficile colitis. The finding in stained smears of 3 to 5 leukocytes in at least 5 high-dry fields suggests colitis and is strongly against the diagnosis of benign or simple antibiotic-diarrhea, but it is not specific for C difficile. A positive test indicates mucosal inflammation and excludes the benign form of antibiotic diarrhea; however, no more than a third of patients with C difficile colitis have positive fecal leukocyte tests.

The isolation of C difficile from stools of a patient with diarrhea does not by itself prove the patient has colitis caused by C difficile, because about 25% of isolates of C difficile obtained from humans are nontoxigenic and nonpathogenic. In addition, at least 3% of healthy adults are asymptomatic carriers off toxigenic isolates of C difficile. Cell culture evidence for cytotoxicity by cytotoxin B is presently the most reliable laboratory aid in diagnosis of pseudomembranous colitis. This test is the "gold standard" used to evaluate all newer tests, but it is expensive, usually takes at least 24 hours, and is not readily available in most hospitals. At least 95 percent of adults with antibiotic-associated diarrhea and toxin B positive stools have colitis.

 

 

54. For the patient in the preceding question, after obtaining diagnostic samples, the most appropriate management would include which of the following?

 

a. Oral metronidazole

b. Intravenous metronidazole

c. Oral vancomycin

d. Intravenous vancomycin

e. Oral bacitracin

Answer: c

 

The efficacy of vancomycin in treatment of PMC has been so well documented that it must be considered the most reliable treatment for the disease. No isolates of C difficile are resistant to vancomycin. Metronidazole, bacitracin, and teicoplanin are alternatives when vancomycin is not tolerated. C difficile isolates are almost always susceptible to vancomycin at concentrations of lower than 5 mg/l, and few isolates have been identified that require more than 16 mg/l. 

Because vancomycin is poorly absorbed into the systemic circulation when given orally at a dosage of 125 to 500 mg four times per day, concentrations far exceeding 16 mg/l, the maximum concentration required for inhibition of C difficile, are easily achieved in stools. Even patients with profuse diarrhea achieve adequate concentrations of vancomycin in stools with these regimens. While small amounts of vancomycin may be absorbed systemically, toxic serum concentrations have never been reported with oral therapy, even when the patient has renal failure.

 

 

55. A 72-year-old woman undergoes anterior resection for a rectal cancer located 7 cm proximal to the anal verge. Pathologic examination of the resected specimen reveals invasion of the tumor into the muscularis propria. Five of 8 lymph nodes contain microscopic tumor. There is no evidence of disseminated disease. Appropriate subsequent management includes which of the following?

 

a. Postoperative radiation plus intravenous 5FU

b. Postoperative radiation alone

c. Observation

d. Postoperative radiation plus intravenous adriamycin

Answer: a

 

In rectal cancer: it is almost as important to prevent local failure and ensuing symptoms as it is to prevent death from distant failure. Radiation therapy is recommended for patients with stage II or III rectal cancers. In a randomized, prospective study, 204 patients with stage II or III rectal cancers were randomized to receive postoperative radiation alone versus radiation plus 5FU and semustine chemotherapy. The group that received chemotherapy had improved local tumor control and increased overall survival. In another prospective study, semustine was found not to be an essential component for effective adjuvant therapy. Based on these and other clinical studies, the NIH has recommended that patients with stage II or III rectal cancers should receive postoperative chemotherapy and radiation as standard care.

 

 

56. Which of the following are tumor suppressor genes that have been associated with the development of colorectal cancer?

 

a. The DCC gene

b. The APC gene

c. The P53 gene

d. The Rb gene

Answer: a, b, c

 

Loss of specific chromosomal regions represent genetic alterations that are associated with the development of colorectal neoplasms in a high percentage of cases. These chromosomal regions have been hypothesized to contain tumor-suppressor genes whose products normally regulate growth and differentiation in a negative fashion. One such gene linked to familial adenomatous polyposis was mapped to the long arm of chromosome 5q and referred to as the APC (adenomatosis polyposis coli) gene. The gene codes for a 300 kilodalton protein which has recently been shown to bind to b-catenin, implying an important role in cell adhesion and possibly cytoskeleton function. It is hypothesized that disruption of cell adhesion and cytoskeleton function can lead to loss of contact inhibition which may promote neoplastic transformation as well as invasiveness of cancer cells.

Another tumor suppressor gene thought to be important in colorectal tumorigenesis is the p53 gene located in chromosome 17p. Alteration in p53 is one of the most common genetic events seen in human malignancies. The p53 gene produces a DNA-binding phosphoprotein that is important in cell proliferation, differentiation and cell survival. Allelic loss of p53 has been observed in more than 75% of colorectal carcinomas.

Another common genetic alteration associated with colorectal tumors is an allelic loss of chromosome 18q. This is where the "deleted in colorectal carcinoma" gene, also termed DCC is located. Mutations in DCC are present in 47% of late adenomas and 73% of carcinomas. The DCC protein shares significant homology with the neural cell adhesion molecule family that regulates cell adhesion and recognition.

 

 

57. Which of the following statement(s) is/are correct with regard to the use of carcinoembryonic antigen (CEA) determinations in management of colorectal cancer?

 

a. CEA determination has 95% specificity when used for screening for colon cancer development in patients with ulcerative colitis

b. CEA levels are increased in 20% of patients with local recurrence after resectional therapy

c. CEA measurements are increased in 90% of patients with disseminated disease

d. CEA levels are increased in 90% of patients with local recurrence after resectional therapy

Answer: b, c

 

CEA is a glycoprotein that was originally described to be a tumor-specific antigen derived from neoplasms of the gastrointestinal tract. CEA is an oncofetal antigen because it is also expressed by early embryonic or fetal cells. It is now known that CEA is not tumor-specific since it can be elevated by a variety of malignancies from different sites as well as in some benign conditions. CEA is not useful as a screening or diagnostic test but is useful as a tumor marker. CEA is elevated in over 90% of patients with disseminated colorectal cancer and in about 20% of patients with localized disease. Serum levels generally are elevated in proportion to the mass of the tumor present and often correlate with response to therapy. CEA levels are useful when elevated levels fall to normal levels after curative resection. In about two thirds of patients with recurrent disease, an increased CEA level is the first indicator of the tumor, and serial CEA testing, combined with regular physical examinations, is one of the most useful tests for detecting recurrent colorectal cancer.

 

 

58. The most common oncogene abnormality observed in association with colorectal cancer is which of the following?

 

a. Overexpression of the N-myc oncogene

b. Amplification of the K-ras oncogene

c. Suppression of the erbB oncogene

d. Amplification of the L-myc oncogene

Answer: b

 

In colon cancer an important genetic alteration that has been demonstrated is a mutation of the K-ras protooncogene. The ras protooncogenes are a family of normal genes (N-ras, H-ras and K-ras) that are highly conserved in nature and encode for the production of GTP-binding proteins (G proteins) which are important for signal transduction. G proteins are involved in the transduction of proliferative signals induced by growth factors or factors involved in cell differentiation. The product of a mutated ras gene is an abnormal G protein which has lost its ability to become inactivated; and thus, results in continuous growth stimulation and autonomous cell growth or differentiation. Experimentally, transfection of normal fibroblasts by mutated ras genes confer neoplastic properties to those cells.

About half of colorectal carcinomas and a similar percentage of adenomas larger than 1 cm in diameter have been found to have the ras gene mutations. By contrast, less than 10% of patients with adenomas smaller than 1 cm have this mutation. It has been postulated that the ras gene mutation may be the initiating event in some colorectal carcinomas or, alternatively, may promote the clonal expansion of a mutated cell population. It appears that the ras gene mutation alone is not responsible for tumorigenesis. Other molecular events appear to be required in addition to ras gene mutations.

 

 

59. Which of the following types of colonic polyps is associated with the highest incidence of malignant degeneration?

 

a. Tubular adenoma

b. Tubulovillous adenoma

c. Villous adenoma

d. Hamartomatous polyp

Answer: c

 

Adenomas can be divided into tubular (75% to 100% tubular component), tubulovillous (25% to 75% villous component), or villous (75% to 100% villous) adenomas. The most common type is tubular adenoma, or adenomatous polyp, which constitutes about 75% of neoplastic polyps. Tubulovillous adenomas represent 15% and pure villous adenomas 10% of neoplastic polyps. All adenomas contain some degree of dysplasia or cellular atypia. This dysplasia can be graded from mild to severe. Carcinoma in situ and severe dysplasia have been grouped together under the classification of high-grade dysplasia. In carcinoma in situ, there is no invasion into the muscularis mucosa, as there is in invasive carcinoma. The incidence of invasive malignancy differs markedly for the three types of adenomas and increases with size. In general, malignancies are seen in 5% of adenomatous polyps, in 22% of tubulovillous adenomas, and in 40% of villous lesions. Hence, although villous lesions are much less common, they are more likely to harbor a malignancy.

 

 

60. A 52-year-old man undergoes a right hemicolectomy for a carcinoma of the ascending colon. Pathological examination of the resected specimen reveals invasion of the tumor to the level of the muscularis propria. Three of 17 lymph nodes contain microscopic tumor. What is the correct Dukes classification (Aster-Coller modification) and associated 5-year survival for this lesion?

 

a. Dukes C2, 45% 5-year survival

b. Dukes B1, 75% 5-year survival

c. Dukes C1, 45% 5-year survival

d. Dukes B3, 65% 5-year survival

Answer: c

 

One of the more commonly used staging systems is the modified Astler-Coller system. According to this system, stage A represents tumors that invade into the mucosa only. Stage B1 tumors invade into but not through the muscularis propria. Stage B2 lesions invade through the bowel wall without adjacent organ involvement, whereas stage B3 tumors involve adjacent organs. Stage C tumors involve regional lymph nodes and are subgrouped into stages C1, C2, and C3, according to depth of bowel wall penetration. Stage D represents evidence of distant organ involvement. In general, the 5-year survival rate for patients with stage D disease is less than 10%. Overall, the 5-year survival rates for stages A, B, and C are 90%, 77%, and 47%, respectively. Additional studies have revealed that among Dukes stage C patients, the number of positive nodes is a significant predictor of survival.

 

 

61. An pedunculated polyp, discovered incidentally at colonoscopy, is removed by snare polypectomy from the ascending colon. Invasive cancer to the level of the submucosa is identified histologically within the polyp. The lesion is well-differentiated. No lymphatic or vascular invasion is noted. The cauterized margin is negative for neoplasm. Appropriate subsequent management includes which of the following?

 

a. Repeat endoscopy at 6 months

b. Right hemicolectomy

c. Subtotal colectomy

d. Repeat endoscopy with fulguration of the polypectomy site

Answer: a

 

With the availability of colonoscopy, endoscopic polypectomy has become the standard approach for the treatment of neoplastic polyps. The risk of this procedure is extremely low, with a complication rate of less than 1%. Almost all pedunculated polyps can be removed endoscopically with a snare. A dilemma in treating colonic polyps occurs when a resected lesion contains a malignant focus. A decision must then be made about the need for a colectomy. If the lesion does not penetrate the muscularis mucosa, it should be considered an in situ malignancy that does not have the propensity to metastasize and, therefore, does not require further surgery. If the lesion penetrates the muscularis mucosa, it is an invasive cancer and may require surgery. In selected cases of pedunculated polyps, conservative management without colectomy may be undertaken if the lesion does not contain poorly differentiated tumor cells or evidence of vascular invasion and if a negative resection margin has been obtained at the level of the stalk. Lesions that are poorly differentiated or have evidence of vascular invasion, regardless of a negative surgical margin, should be treated by colectomy.

 

 

62. Dietary risk factors thought to play a causative role in development of colorectal cancer include which of the following?

 

a. High fat intake

b. Low fiber intake

c. High smoked food intake

d. High vegetable intake

Answer: a, b

 

Evidence from epidemiological studies suggest that dietary factors play important causative roles in the development of large bowel cancers. From these studies, fat intake has been the most consistently positive association and fiber intake the most consistently inverse association noted in the incidence of colorectal cancer. In comparisons between countries, the rates of colon cancer are strongly associated with the intake of animal fat and meat. The associations between per capita consumption of total fat, saturated fat, and cholesterol and national incidence rates of colon cancer are strongly positive. The proposed mechanism by which dietary fat may increase the risk of developing colonic cancer is by its interaction with bile acids.

The relation between fiber intake and colon cancer was initially noted by Burkitt, who reported low rates of colon cancer in areas of Africa where fiber consumption and stool bulk were high. In general, epidemiological studies have demonstrated that fiber intake is higher in non-western countries with lower incidence of colon cancer. The role of fiber was originally seen simply as providing bulk to dilute potential carcinogens and speed their transit through the colon. This appears to be an oversimplification and the relation between fiber intake and colon cancer is more complex. Additional studies have suggested that certain fibers may bind mutagens, which reduces their contact with colonic epithelium, may favorably change the fecal pH, and may participate in other complex interactions.

 

 

63. Which of the following statements with regard to resection of rectal cancers is/are true?

 

a. A distal margin of 5 cm should be obtained because 42% of patients have microscopic evidence of intramural spread beyond 3 cm from the palpable tumor

b. A distal margin of 3 cm should be obtained because only 3% of patients have microscopic evidence of intramural spread beyond 2 cm from the palpable tumor

c. Local recurrence rates correlate strongly with distal margins less than 4 cm

d. There is no correlation between local recurrence and distal margins beyond 2 cm

Answer: b, d

 

One of the controversies surrounding sphincter-saving procedures for rectal tumors is the length of adequate distal mucosal margin. The traditional dictum of 5 cm for a margin is not substantiated by any studies. Only 2.5% of patients have intramural spread beyond 2 cm from the palpable tumor, and these patients generally have dissemination of tumor despite aggressive local therapy. There is no correlation between local recurrence and the extent of distal margin when it is greater than 2 cm. Ideally, a surgical margin of 3 cm, measured on the fresh specimen, should be achieved.

 

 

64. A 58-year-old male undergoes resection of a Dukes C2 colon cancer via right hemicolectomy. Three years postoperatively, rising CEA levels prompt evaluation including abdominal computed tomography. Two lesions, each measuring 2 cm, are noted in the right hepatic lobe. No other abnormalities are noted. A right hepatic lobectomy is performed without complication. Which of the following most closely approximates anticipated 5-year survival?

 

a. 85–90%

b. 65–70%

c. 45–50%

d. 25–30%

Answer: d

 

The liver is the most frequent site of blood-borne metastases from primary colorectal cancers. In a subgroup of patients, the liver may be the only site of recurrent disease, and surgical excision of these metastases is the only curative option for these patients. Overall, surgical resection is associated with a 25% to 30% 5-year survival rate. Patients eligible for hepatic resection of metastatic disease are those who have no evidence of extrahepatic tumor, no medical contraindications for surgery, and less than four lesions that are amenable to resection with negative surgical margins.

 

 

65. An asymptomatic 52-year-old man is undergoing screening sigmoidoscopy. In the rectum, at 6 cm from the anal verge, a 2 cm yellow, submucosal nodule is noted. Deep endoscopic biopsies are consistent with carcinoid. Appropriate management includes which of the following?

 

a. Observation

b. Transanal excision

c. Low anterior resection

d. Abdominoperineal resection

Answer: b

 

Most gastrointestinal tract carcinoids occur in the ileum and the appendix. The rectum is the next most common site, and occasional carcinoid tumors occur in the colon. Tumor size is an extremely important prognostic factor. About 60% of rectal carcinoids present as asymptomatic submucosal nodules measuring less than 2 cm in diameter. Transanal local excision suffices for definitive therapy since small tumors rarely metastasize. Malignant potential is seen almost exclusively in patients with tumors larger than 2 cm. More radical excisions of larger rectal lesions may be required for local control; however, the results of radical excisions for large rectal carcinoids are poor since they are prone to metastasize.

 

66. A 72-year-old woman complains of anal itching and burning. Physical examination reveals an erythematous, scaly lesion, 3 cm in circumference, within the anal canal. The intersphincteric groove can not be appreciated in the area of the lesion. The remainder of the physical examination is normal. Appropriate initial management includes which of the following?

 

a. Acyclovir 200 mg QID for 10 days

b. Hydrocortisone cream 0.1% topically for 14 days

c. Incisional biopsy

d. Metronidazole 250 mg PO QID for 14 days

Answer: c

 

Extramammary Paget’s disease may be found in the axilla and in the anogenital region, including the labia majora, penis, scrotum, groin, pubic area, perineum, perianal region, thigh, and buttock. Paget’s disease of the perianal area is a malignant neoplasm of the intraepidermal portion of apocrine glands with or without associated dermal involvement. Paget’s disease has a long preinvasive phase, but if untreated, an invasive adenocarcinoma of the apocrine gland type develops. The disease is more common in women than men, with the highest incidence in the seventh decade.

Macroscopically, the lesion appears as an erythematous scaly or eczematoid plaque-like lesion, similar to other benign perianal lesions, making clinical diagnosis difficult. A definite diagnosis is made by biopsy, which shows characteristic histologic appearance—large, pale, vacuolated cells with hyperchromatic eccentric nuclei. The cells invariably contain acid mucosubstances, an important feature in distinguishing this lesion from melanoma and Bowen’s disease.

 

 

67. For the patient in the preceding question, biopsy revealed an invasive apocrine gland neoplasm. The deep margins included striated muscle infiltrated by neoplastic cells. Appropriate management includes which of the following?

 

a. Primary radiation

b. Abdominoperineal resection with bilateral inguinal lymph node dissection

c. Abdominoperineal resection only

d. Carbon dioxide laser fulguration

Answer: c

 

Wide local excision is the treatment of choice in the absence of invasive carcinoma. Because of the high incidence of local recurrence and residual tumor, it is vitally important to obtain an adequate resectional margin. Grossly, the extent of involvement is ill defined, and multiple punch biopsies may be required to determine the extent of involvement. For more advanced lesions with underlying carcinoma, an abdominoperineal resection is indicated. Inguinal lymph node dissection is performed only if groin lymph nodes are clinically positive for metastasis. Because of the commonly delayed diagnosis (average, 4 years), about 25% of patients with perianal Paget’s disease have metastases when they seek treatment. The sites of metastases, in order of frequency, are inguinal and pelvic lymph nodes, liver, bone, lung, brain, bladder, prostate, and adrenal gland. The prognosis is poor once metastasis has occurred.

 

 

68. A 43-year-old woman presents with complaints of anal pain and spotting of blood with defecation. Physical examination reveals a 2  3 cm area of ulceration within the anal canal. The remainder of the physical examination is normal. Incisional biopsy is positive for squamous cell carcinoma. Appropriate management includes which of the following?

 

a. Abdominoperineal resection

b. Wide local excision, skin grafting, proximal diverting colostomy

c. Primary radiation therapy

d. Local excision and primary closure

Answer: c

 

For localized squamous cell cancers of the anal canal, the most effective protocol consists of primary irradiation and chemotherapy. The treatment regimen includes:

 

1. External irradiation, 3000 rad (30 Gy), to the primary tumor, pelvic, and inguinal nodes from day 1 to day 21 (200 rad/d, 5 days a week)

2. Systemic chemotherapy, 5-fluorouracil at 1000 mg/m2/24 h, as a continuous infusion for 4 days, starting on day 1 of radiotherapy and repeated on days 28 through 31

3. Mitomycin C at 15 mg/m2 intravenous bolus on day 1

 

If the lesion disappears grossly, and its microscopic absence is confirmed by biopsy, no further treatment is necessary.

 

69. Recurrent episodes of sigmoid colonic diverticulitis prompt operative therapy. Which of the following describe the appropriate margins for resection?

 

a. Proximal margin, splenic flexure; distal margin, rectosigmoid junction

b. Proximal margin, descending colon; distal margin, rectosigmoid junction

c. Proximal margin, descending colon; distal margin, mid-rectum

d. Proximal margin, transverse colon; distal margin, mid-rectum

Answer: b

 

At the time of exploratory laparotomy, if the disease is localized, a segmental colectomy should be performed. The distal extent of the resection should always extend to the proximal rectum to decrease the chance of recurrence. The proximal extent of resection should include the segment involved with the acute disease plus any additional colon with signs of chronic disease or large numbers of diverticula. With this approach, the recurrence rate after surgical resection is less than 10%. The only absolute contraindications to primary anastomosis are free perforation with generalized peritonitis; obstruction with unprepared bowel; and intraoperative conditions that do not warrant primary anastomosis, such as septic shock, ureteral injury, or other medical conditions that make a prolonged operation inadvisable. If resection is thought to be unsafe in the presence of a massive phlegmon or if the patient is too unstable to undergo a resection, a diverting end colostomy with mucous fistula may be appropriate, with planned colonic resection at a later date after inflammation subsides.

 

 

70. An elderly man presents with complaints that he is passing gas with urination. The past medical history is positive for one episode of diverticulitis, treated medically, transurethral resection of the prostate for benign prostatic hypertrophy, and diabetes. Which of the following diagnostic tests is most appropriate initially?

 

a. Computed tomography of the abdomen and pelvis

b. Cystoscopy

c. Barium enema

d. Intravenous pyelography

Answer: a

 

Colovesical fistulas account for about half of fistulas due to diverticulitis. Most patients with colovesical fistula present with urinary tract symptoms, including urgency, dysuria, pneumaturia, and fecaluria. In spite of obvious symptoms, the diagnosis of colovesical fistula can be difficult to establish conclusively. Recurrent urinary tract infections in an elderly man should increase suspicion. Barium enema usually demonstrates diverticula, or occasionally, sigmoid narrowing. Only rarely is the fistulous tract actually filled. Cystoscopy demonstrates hyperemia and inflammation consistent with chronic cystitis. Although these findings may be localized to some extent, indicating the presence of a fistulous communication, the fistulous opening is seldom seen. CT using intraluminal contrast material has emerged as the most sensitive test for the presence of a colovesical fistula. The presence of barium in the urine is diagnostic of a colovesical fistula. In over 90% of patients, air is visualized in the urinary bladder, and an indurated segment of sigmoid colon is observed adjacent to a locally thickened bladder wall.

 

 

71. For the patient in the preceding question, a colovesical fistula originating from the sigmoid colon is demonstrated. Colonoscopy reveals diverticula and excludes carcinoma. During laparotomy, a thickened sigmoid colon is found to be adherent to the dome of the bladder. A definite fistula is not observed. Appropriate operative management includes which of the following?

 

a. Sigmoid resection, primary colonic anastomosis, catheter drainage of bladder

b. En-bloc resection of sigmoid colon and adjacent bladder wall, primary colonic anastomosis, suprapubic cystostomy

c. En-bloc resection of sigmoid colon and adjacent bladder wall, formation of descending colostomy and Hartmann’s pouch, suprapubic cystostomy

d. Sigmoid resection, primary colonic anastomosis, bilateral percutaneous nephrostomies

Answer: a

 

Most patients with colovesical fistulas are effectively treated by a one-stage procedure consisting of segmental colectomy and closure of the fistulous opening in the bladder. The proximal margin of resection should include the entire segment of thickened, contracted colon and any additional colon that is involved in the acute inflammation. If the fistulous opening is not identifiable, implying that it is small in diameter, nothing needs to be done to identify the bladder fistula. Urinary catheter drainage for 7 to 10 days, followed by cystographic verification of closure of the fistula, is sufficient therapy. Depending on the severity of the related complications of diverticulitis (obstruction, inflammation, abscess, sepsis, other fistulas), it may occasionally be necessary to perform a two-stage procedure, the first stage being segmental colectomy and colostomy formation and the second stage consisting of closure of the colostomy. Either the one-or two-stage procedure can be done with low morbidity and mortality and with less than a 5% recurrence rate.

 

 

72. A 65-year-old woman develops obstipation, lower abdominal pain, and fever. Physical examination reveals a temperature of 38.5°C, left lower quadrant tenderness, and an ill-defined lower abdominal mass. White blood count is 17,500 per mm3. Intravenous hydration, broad spectrum antibiotics, and analgesics are ordered. After 48 hours, symptoms have not improved. Appropriate management includes which of the following?

 

a. Barium enema

b. Computed tomography of the abdomen

c. Immediate laparotomy

d. Intravenous pyelogram

Answer: b

 

Signs and symptoms of diverticulitis include fever, tachycardia, leukocytosis with left shift of the differential count, abdominal pain, and a tender lower abdominal mass. Most patients with an acute episode of diverticulitis can be treated with intravenous fluids, bowel rest, broad-spectrum antibiotics, and analgesics.

If the patient does not improve within 48 hours, complications of diverticulitis may exist, and further investigation is necessary. Only about 20% of patients develop complications of diverticulitis with their first episode; this rises to 60% with recurrent episodes. Although a water-soluble contrast enema radiograph can provide the diagnosis of diverticulitis, computed tomography (CT) has become the preferred diagnostic test in patients that do not improve within 48 hours. CT is especially useful in delineating the complications of diverticulitis, including perforation and abscess formation. In addition, at the time of CT scan, percutaneous drainage catheters can be placed if an abscess is identified. After CT drainage of an abscess, 50% to 90% of patients may undergo a successful one stage procedure of segmented colectomy and primary anastomosis. If percutaneous drainage is not feasible, or an abscess is not identified, surgical intervention is recommended.

 

 

73. Which of the following statement(s) relating to anal sphincteric function is/are correct?

 

a. When the rectum is distended, the internal anal sphincter relaxes and the external anal sphincter contracts

b. When the rectum is distended, the internal anal sphincter contracts and the external anal sphincter relaxes

c. The external anal sphincter is responsible for resting anal pressure

d. The internal anal sphincter is responsible for resting anal pressure

Answer: a, d

 

The internal sphincter, because it is innervated by the autonomic nervous system, is not subject to voluntary control. This powerful muscle exists in a continuously tonic state, and is responsible for maintaining closure of the resting anal canal. The high-pressure zone of the anal canal at rest is due to the actions of the internal sphincter. The external sphincter contributes to anal pressure only when a bolus of stool is present within the anal canal. The increase in pressure during voluntary contraction (squeeze pressure) is exclusively due to the activity of the external sphincter. The high resting pressure in the anal canal acts as a barrier to prevent leakage of mucus and gas.

When the rectum is distended, the internal sphincter relaxes. This relaxation allows the rectal content to move down to the anal canal. In contrast, when the rectum is distended, the external sphincter contracts. Reflex contraction of the external sphincter prevents rectal content from leaking through the anus. Although volitional contraction of the external sphincter can only be sustained for short periods, it is the most important mechanism of voluntary continence.

 

 

74. The most common complication after hemorrhoidectomy is which of the following?

 

a. Urinary retention

b. Rectal bleeding

c. Incontinence

d. Wound infection

Answer: a

 

Hemorrhoidectomy should be considered when the hemorrhoids are severely prolapsed through the anus, requiring manual replacement, or in hemorrhoids complicated with associated pathology, such as ulceration, fissure, fistula, large hypertrophied anal papilla, or extensive skin tags. An elliptic excision starts at the perianal skin, includes external and internal hemorrhoids, and ends at the anorectal ring. The mucosa and submucosa are dissected off the underlying internal sphincter muscle. Unless there is an associated anal stenosis or chronic anal fissure, internal sphincterotomy is not performed. The entire wound is closed with running absorbable suture. The largest and the most redundant hemorrhoid should be excised first. No packing is placed in the anal canal. Urinary retention is the most common complication of hemorrhoidectomy, and can be avoided if intravenous fluids are restricted during the procedure and minimized for the next 6 to 8 hours.

 

 

75. Appropriate treatment of chlamydial proctitis includes which of the following?

 

a. Tetracycline 500 mg QID

b. Metronidazole 250 mg QID

c. Acyclovir 200 mg QID

d. Erythromycin 500 mg QID

Answer: a, d 

C trachomatis is the most common cause of sexually transmitted disease in the United States, affecting 4 million Americans each year. Proctoscopy reveals a picture of nonspecific proctitis with friable, granular, and edematous mucosa. Immunofluorescent microscopy provides an accurate and a rapid diagnosis. Treatment includes tetracycline hydrochloride, 500 mg by mouth four times daily for 7 days, or doxycycline, 100 mg by mouth twice daily for 7 days. For patients in whom tetracyclines are contraindicated, erythromycin base or stearate, 500 mg by mouth four times daily for 7 days, or erythromycin ethylsuccinate, 800 mg by mouth four times daily for 7 days, may be used. Two new drugs have been approved by the FDA for the treatment of chlamydia—Azithromycin, 1 gm orally in a single dose, and Ofloxacin, 300 mg orally two times a day for 7 days. A substantial advantage of Azithromycin, in comparison with all other therapies, is that a single dose is effective; this antimicrobial may prove most useful in situations in which compliance with a seven day regimen of another antimicrobial cannot be ensured. In view of the high efficacy of tetracycline and doxycycline, cost also should be considered when selecting a treatment regimen.

 

76. A 65-year-old man presents with complaints of mucous discharge and perianal discomfort. Physical examination reveals a fistulous opening lateral to the anus. Anoscopic examination permits passage of a probe through the fistula tract. The fistula traverses the internal anal sphincter, the intersphincteric plane, and a portion of the external anal sphincter. The fistula is categorized as which type? 

a. Intersphincteric

b. Transsphincteric

c. Suprasphincteric

d. Extrasphincteric

Answer: b

 

There are four main forms of fistula-in-ano, based on the relation of the fistula to the sphincter muscles. An intersphincteric fistula tract is in the intersphincteric plane. The external opening is usually in the perianal skin close to the anal verge. A transsphincteric fistula starts in the intersphincteric plane or in the deep postanal space. The fistulous track traverses the external sphincter, with the external opening at the ischioanal fossa. Horseshoe fistulas are in this category. Suprasphincteric fistulas start in the intersphincteric plane in the mid-anal canal and then pass upward to a point above the puborectalis. The fistula passes laterally over this muscle and downward between the puborectalis and the levator ani muscle into the ischioanal fossa. An extrasphincteric fistula passes from the perineal skin through the ischioanal fossa, the levator ani muscle, and finally penetrates the rectal wall. Extrasphincteric fistulas may arise from cryptoglandular origin, trauma, foreign body, or pelvic abscess.

 

77. For the patient in the preceding question, appropriate management includes which of the following? 

a. Division of the tissues over the probe with electrocautery, leaving the wound open to heal by secondary intention

b. Division of the tissues over the probe with electrocautery, closing the wound using a pedicled skin flap

c. Division of the internal anal sphincter using electrocautery, encircling the external sphincter with a seton

d. Proximal diverting colostomy and antibiotics

Answer: c 

In young patients, transection of internal and external sphincter muscles in the posterior half, when performed in the course of a fistulotomy, does not always jeopardize anal continence. In older patients and in women, however, transection of the external sphincter muscle, particularly in the anterior half, risks incontinence. When external sphincter transection appears likely, some authors recommend the use of a seton. A seton is a suture that is drawn through a fistula. The rationale for using a seton is to create fibrosis. The seton is threaded through the fistulous track and tied over the muscles. In the second stage (average interval, 6 to 8 weeks), fistulotomy is performed. Incontinence after the proper use of seton is uncommon, even when the fistula is deep.